הבדלים בין גרסאות בדף "משפט לייבניץ"

מתוך Math-Wiki
קפיצה אל: ניווט, חיפוש
(יצירת דף עם התוכן "חזרה למשפטים באינפי ==משפט לייבניץ לטורים עם סימנים מתחלפים== תהי <math>a_n</math>...")
 
(הוכחה)
שורה 9: שורה 9:
  
 
===הוכחה===
 
===הוכחה===
 +
נוכיח כי סדרה הסכומים החלקיים של הטור הינה סדרת קושי, ועל כן הטור מתכנס.
 +
 +
יהי אפסילון גדול מאפס, צריך למצוא מקום בסדרה שהחל ממנו והלאה ההפרש בין כל שני איברים קטן מאפסילון.
 +
 +
*<math>|S_m-S_n|=|(-1)^ma_m+...+(-1)^{n+1}a_{n+1}|=|a_m-a_{m-1}+a_{m-2}-...| </math>
 +
 +
נראה כי כל איבר "בולע" את קודמיו, לפי המונוטוניות של הסדרה:
 +
 +
::<math>-a_{m-1}<a_m-a_{m-1}<0</math>
 +
 +
לכן
 +
 +
::<math>a_{m-2}-a_{m-1}<a_m-a_{m-1}+a_{m-2}<a_{m-2}+0</math>
 +
 +
כלומר
 +
 +
::<math>0<a_m-a_{m-1}+a_{m-2}<a_{m-2}</math>
 +
 +
וכן הלאה עד שנקבל
 +
 +
::<math>|S_m-S_n|<a_{n+1}</math>
 +
 +
 +
וכיוון ש<math>a_n</math> שואפת לאפס, החל ממקום מסויים זה קטן מאפסילון (ללא תלות ב-m).
 +
 +
 +
לפי טיעון דומה, <math>|\sum_{n=k+1}^K (-1)^na_n|=|a_{k+1}-a_{k+2}+a_{k+3}-...|\leq a_{k+1}</math> ולכן
 +
 +
::<math>|R_k|=\lim_{K\rightarrow \infty}|\sum_{n=k+1}^K (-1)^na_n|\leq a_{k+1}</math>
 +
 +
כפי שרצינו.

גרסה מ־10:37, 2 בפברואר 2012

חזרה למשפטים באינפי

משפט לייבניץ לטורים עם סימנים מתחלפים

תהי a_n סדרה חיובית, מונוטונית, השואפת לאפס. אזי:

  • הטור \sum_{n=1}^\infty (-1)^na_n מתכנס
  • השארית R_k=\sum_{n=1}^\infty (-1)^na_n-\sum_{n=1}^k (-1)^na_n מקיימת |R_k|\leq a_{k+1}

הוכחה

נוכיח כי סדרה הסכומים החלקיים של הטור הינה סדרת קושי, ועל כן הטור מתכנס.

יהי אפסילון גדול מאפס, צריך למצוא מקום בסדרה שהחל ממנו והלאה ההפרש בין כל שני איברים קטן מאפסילון.

  • |S_m-S_n|=|(-1)^ma_m+...+(-1)^{n+1}a_{n+1}|=|a_m-a_{m-1}+a_{m-2}-...|

נראה כי כל איבר "בולע" את קודמיו, לפי המונוטוניות של הסדרה:

-a_{m-1}<a_m-a_{m-1}<0

לכן

a_{m-2}-a_{m-1}<a_m-a_{m-1}+a_{m-2}<a_{m-2}+0

כלומר

0<a_m-a_{m-1}+a_{m-2}<a_{m-2}

וכן הלאה עד שנקבל

|S_m-S_n|<a_{n+1}


וכיוון שa_n שואפת לאפס, החל ממקום מסויים זה קטן מאפסילון (ללא תלות ב-m).


לפי טיעון דומה, |\sum_{n=k+1}^K (-1)^na_n|=|a_{k+1}-a_{k+2}+a_{k+3}-...|\leq a_{k+1} ולכן

|R_k|=\lim_{K\rightarrow \infty}|\sum_{n=k+1}^K (-1)^na_n|\leq a_{k+1}

כפי שרצינו.